Download as docx, pdf, or txt
Download as docx, pdf, or txt
You are on page 1of 53

1

Constitution of India 2

Unit-I

Qs. President of India, Council of Ministers and their relationship; and powers of President
– pardoning power and ordinance making power.

Ans. Introduction -

In India, the constitution establishes a Parliamentary form of Government. The essence of the
parliamentary type of government is that the head of the State is Constitutional head and the real
executive powers are vested in the Council of Ministers. The Prime Minister is the head of the
Council of Ministers. The Council of Ministers is responsible to the House of the People. Article
52 of the Indian Constitution says that there shall be a President of India. He is the head of the
State. According to Article 53, The executive pankaj
power of the union shall be vested in the President
and it shall be exercised by him in accordance with the Constitution either directly or through
officers subordinate to him. The expression "offices Subordinate to him" includes Prime
Ministers also. He has a right to stay in Rashtrapati Bhavan, that is his official residence, without
payment of rent. The president enjoys personal immunity against any liability for his official acts
during his term of office. The procedure of each removal is prescribed in Article 61 of the Indian
Constitution. This Quasi-judicial procedure is also called as impeachment. The President takes
oath in Presence of the Chief Justice of India in absence of him, before the senior most judge of
the Supreme Court According to Article 60.

The President of India

Article 52 of the Indian Constitution says that, there shall be a President of India.

Election of President (Article 54)

The President shall be elected by the members of an electoral college consisting of


(a) the elected members of both Houses of Parliament; and

(b) the elected members of the Legislative Assemblies of the States.

Term of office of President (Article 56)

The President shall hold office for a term of five years from the date on which he enters upon his
office

Provided that –

(a) the President may, by writing under his hand addressed to the Vice-President, resign his
office;
2

(b) the President may, for violation of the Constitution, be removed from office by impeachment
in the manner provided in article 61;

(c) the President shall, notwithstanding the expiration of his term, continue to hold office until
his successor enters upon his office.

Any resignation addressed to the Vice-President under clause (a) of the proviso to clause (1)
shall forthwith be communicated by him to the Speaker of the House of the People.

Eligibility for re-election

Article 57 of the Indian Constitution says that a person who holds, or who has held, office as
President shall, subject to the other provisions of this Constitution, be eligible for re-election to
that office.
pankaj
Qualifications for election as President

According to Article 58 of the Indian Constitution, no person shall be eligible for election as
President unless he –

(a) is a citizen of India,

(b) has completed the age of thirty-five years, and

(c) is qualified for election as a member of the House of the People.

A person shall not be eligible for election as President if he holds any office of profit under the
Government of India or the Government of any State or under any local or other authority
subject to the control of any of the said Governments.

Explanation - For the purposes of Article 58, a person shall not be deemed to hold any office of
profit by reason only that he is the President or Vice-President of the Union or the Governor of
any State or is a Minister either for the Union or for any State.

Oath or affirmation by the President (Article 60)

Every President and every person acting as President or discharging the functions of the
President shall, before entering upon his office, make and subscribe in the presence of the Chief
Justice of India or, in his absence, the senior-most Judge of the Supreme Court available, an oath
or affirmation in the following form, that is to say –

"I, A.B., do swear in the name of God solemnly affirm that I will faithfully execute the office of
President (or discharge the functions of the President) of India and will to the best of my ability
preserve, protect and defend the Constitution and the law and that I will devote myself to the
service and well-being of the people of India".
3

Procedure for impeachment of the President (Article 61)

(1) When a President is to be impeached for violation of the Constitution, the charge shall be
preferred by either House of Parliament.

(2) No such charge shall be preferred unless -(a) the proposal to prefer such charge is contained
in a resolution which has been moved after at least fourteen days' notice in writing signed by not
less than one-fourth of the total number of members of the House has been given of their
intention to move the resolution, and(b) such resolution has been passed by a majority of not less
than two-thirds of the total membership of the House.

(3) When a charge has been so preferred by either House of Parliament, the other House shall
investigate the charge or cause the charge to be investigated and the President shall have the right
to appear and to be represented at such investigation.
pankaj
(4) If as a result of the investigation a resolution is passed by a majority of not less than two-
thirds of the total membership of the House by which the charge was investigated or caused to be
investigated, declaring that the charge preferred against the President has been sustained, such
resolution shall have the effect of removing the President from his office as from the date on
which the resolution is so passed.

Council of Ministers

The Union Council of Ministers exercises executive authority in the Republic of India. It consists
of senior ministers, called 'cabinet ministers', junior ministers, called 'ministers of state' and,
rarely, deputy ministers.

It is led by the Prime Minister. A smaller executive body called the Union Cabinet is the supreme
decision-making body in India. Only the prime minister and ministers of the rank of cabinet
minister are members of Union Cabinet in accordance with Article 75

Pursuant to Article 75(3), the Council of Ministers is responsible collectively to the lower house
of the Indian parliament, called the Lok Sabha (House of the People). When a bill introduced by
a minister in the Lok Sabha is not approved by it, the entire council of ministers is responsible
and not the minister. The council of ministers upon losing confidence of Lok Sabha shall resign
to facilitate new government formation.

A minister shall not take any decision without being considered by the council of ministers
per Article 78(c). All union cabinet members shall submit in writing to the President to propose
proclamation of emergency by the president in accordance with Article 352.

According to the Constitution of India, the total number of ministers in the council of ministers
must not exceed 15% of the total number of members of the Lok Sabha. Ministers must be
4

members of parliament. Any minister who is not a member of either of the houses of the
parliament for six consecutive months is automatically stripped off his or her ministerial post.

Ranking

There are five categories of the council of ministers as given below, in descending order of rank:

I. Prime Minister
II. Deputy prime minister (if any); presides as prime minister in his absence or as the senior
most cabinet minister.
III. Cabinet minister: member of cabinet; leads a ministry.
IV. Minister of state (independent charge): junior minister not reporting to a cabinet minister.
V. Minister of state (MoS): deputy minister reporting to a cabinet minister, usually tasked
with a specific responsibility in that ministry.
pankaj
Appointment

Pursuant to Article 75, a minister who works at the pleasure of president, is appointed by the
president on the advice of the prime minister.

Removal

I. Upon death
II. Upon self resignation
III. Upon dismissal by the President for minister's unconstitutional acts per Article 75(2).
IV. Upon direction from the Judiciary for committing violation of law.
V. Upon ceasing eligibility to be a member of Parliament.

Council of Ministers of a State

Every state in India is governed by its council of ministers with rules and procedures similar to
union council of ministers per Articles 163, 164 and 167(c). There is no concept of cabinet rank
minister in the council of ministers.

Ordinance making power of the President

Introduction

According to Article 52 of the Indian Constitution, there shall be a President of India. He is the
head of the State. The Constitution of India confers certain Power to the president of India. It
includes executive power, Legislative powers, Veto powers, Financial powers, Judicial
powers, Military and Diplomatic powers, Emergency powers, Pardoning power and Ordinance
making power. In no country, except India, the executive is vested with legislative power. The
Indian Constitution expressly confers power to make Ordinance on the President. Power to make
5

ordinance is justified on the ground that the president must be armed with powers to meet with
the serious situation when the Houses of Parliament are not in

session.
What is ordinance?

An ordinance is a temporary law having the force of law. When both the houses of the
Parliament are not in session, the president can promulgate Ordinance under article 123 of the
Indian Constitution.

Ordinance making power of the president

The most Important legislative power of the President is his Ordinance making Power. If at any
time, when both houses of the Parliament are not in session and the President is satisfied that
circumstances exist which render it necessarypankaj
for him to take immediate action, he may issue
such ordinance at the circumstances appear to him to require. The Ordinance issued by President
shall have the same force as an Act of Parliament. An ordinance ceases to operate on the expiry
of six weeks from the reassembly of parliament. In case if the houses reassemble on a different
date, the period of six weeks is calculated from the later date. Without parliamentary approval,
an ordinance can last for a maximum period of 6 months and 6 weeks. all acts done completed
under an unapproved ordinance will lapse.
The President may withdraw this ordinance at any time. The president exercises this power on
the advice of the Prime Minister headed by the Council of Ministers. an ordinance cannot be
used to amend the Constitution

Pardoning Power

The President is empowered under Article 72 of the Constitution to grant pardon to a Convict in
case of offense against Union law, sentenced or punished by Court martial, sentenced do Death.

Pardon :

Pardon means to have absolve the Convict completely.

Commutation

to replace the punishment with a less severe punishment.

Remission

to reduce the period of sentence, but the character of punishment remains same.

Respite

a lesser punishment taking into consideration a special case like a pregnant woman Convict.
6

Reprieve

to stay the punishment temporarily.

Qs. Governor – Position and powers including powers of Governor under J & K
Constitution.

Ans. The State Executive consists of the Governor, the State Council of Ministers and the
Advocate General of the State.
Article 153 of the Constitution of India provides that, "there shall be a Governor for each State".
The Governor of a state is appointed by the President of India. There can be a single person as
the Governor for two or more states. The powers and functions of a Governor at State level is
similar to that of a President at the Union level.pankaj
Governor is the Constitutional or nominal head of the State Executive and is bound by the advice
of the State Council of Ministers.
Article 155 says that the Governor is appointed by the President of India by warrant under his
hand and seal.

Qualifications
Article 157 provides for the required qualifications for a Governor. In order to be eligible for
appointment of Governor -

 He must be a citizen of India


 He must have completed 35 years of age.

Terms of Office
Article 156(1) provides that a Governor holds office during the pleasure of the President. He may
by writing to the President under his hand resign his Office. Thus, a Governor holds office for a
term of five years from the date on which he enters upon his office.

Powers of the Governor


The powers and functions of the Governor can be explained under the following heads.

 Executive Powers
 Financial Powers
 Legislative Powers
 Ordinance Making Powers
 Power to grant pardons
Executive Powers
Article 154(1)
7

The Executive power of the State is vested in the Governor. The Governor is the constitutional
head or nominal head of the State and the real executive power lies in the State Council of
Ministers. All executive actions of the State shall be expressed to be taken in the name of the
Governor.
The Governor appoints the Chief Minister and on his advice the State Council of Ministers. The
Chief Minister holds the office on the pleasure of Governor, but in real sense, he holds the office
as long as he enjoys the support of the majority of the Vidhan Sabha. On the advice of the Chief
Minister, the Governor appoints the State Council of Ministers and distributes the portfolios
among them.
The Governor also appoints the Advocate General of the State, the chairman and members of the
State Public Service Commission. He also appoints the district court judges.
The President of India consults the Governor on appointment of the High Court judges.
Article 166 provides the following.
pankaj
(1) All executive action of the Government of a State shall be expressed to be taken in the name
of the Governor
(2) Orders and other instruments made and executed in the name of the Governor shall be
authenticated in such manner as may be specified in rules to be made by the Governor, and the
validity of an order on instruction which is so authenticated shall not be called in question on the
ground that it is not an order or instrument made or executed by the Governor
(3) The Governor shall make rules for the more convenient transaction of the business of the
Government of the State, and for the allocation among Ministers of the said business in so far as
it is not business with respect to which the Governor is by or under this Constitution required to
act in his discretion
Financial Powers
Articles 202 and 203 provides for the financial powers of the Governor.
Money Bill cannot be introduced in the Legislature Assembly of the State without the prior
recommendation of the Governor. No demands for grants can be made except on the
recommendation of the Governor. Article 202 says that a Governor also causes to be laid the
annual financial statement or the State Budget before the State Legislature.
The Governor constitutes the State Finance Commission and he is entitled to draw advances out
of the State Contingency Fund to meet any unforeseen expenses by the State.
Legislative Powers
The Governor summons the Houses or each House of the Legislature to meet at such time and
place as he thinks fit. However, six months must not lapse between the last sitting of the House
in the prior session and the first sitting of the next session.
The Governor has the power to dissolve the Legislative Assembly. However these powers are
formal and the Governor must act according to the advice of the Chief Minister and his Council
of Ministers. A bill that a State Legislature has passed can only become a law only if it is
approved by the Governor. However if the Governor is not satisfied with the contents of the bill
8

he can return it to the State Legislature. However, he must assent it on the second time. The
Governor has the right to reserve certain bills for the assent of the President.
Ordinance Making Power
One of the most important legislative power the Governor holds is his Ordinance making power.
Under Article 213, when the Legislature is not in session and if the Governor is satisfied that
circumstances exist which require him to take immediate action, he may legislate by Ordinance.
The following two conditions are to be satisfied.

 When the Legislative Assembly of the State is not in Session or where there are two Houses
in the Legislature, when both Houses are not in Session
 The Governor must be satisfied that circumstances exist which render it necessary for him to
take immediate action.
An Ordinance promulgated under Article 213 shall have the same force and effect as an Act of
the Legislature of the State. The Ordinance expires
pankaj after six weeks of the reassembly of the
Legislature unless it is approved by the Legislature.
The Ordinance may be withdrawn at any time by the Governor.
Power to grant Pardons
Article 161 confers on the Governor, the power to grant pardons, reprieves or remission of
punishment or to suspend, remit or commute the sentence of any person convicted of any offence
against any law relating to which the executive power of the State extends.
A similar power is conferred on the President via Article 72. The main differences being the
President can grant pardon in cases of death sentence and the President can pardon punishments
of sentences inflicted by Court Martial.
Qs. Legislative privileges and the Problem of Codification.
Ans. Introduction

Parliamentary privileges are defined in Article 105 of the Indian Constitution. The members of
Parliment are exempted from any civil or criminal liability for any statement made or act done in
the course of their duties. The privileges are claimed only when the person is a member of the
house. As soon as he ends to be a member, the privileges are said to be called off. The privileges
given to the members are necessary for exercising constitutional functions. These privileges are
essential so that the proceedings and functions can be made in a disciplined and undisturbed
manner.

The privileges individually enjoyed by the members are

Freedom of speech in parliament

The members of the parliament have been vested with the freedom of speech and expression. As
the very essence of our parliamentary democracy is a free and fearless discussion, anything said
by them expressing their views and thoughts are exempted from any liability and cannot be tried
in the court of law.
9

The freedom of speech and expression guaranteed to a citizen under Article 19(2) is different
from the freedom of speech and expression provided to a member of the parliament. It has been
guaranteed under Article 105(1) of the Indian constitution. But the freedom is subject to rules
and orders which regulates the proceedings of the parliament.

This right is given even to non-members who have a right to speak in the house. Example,
attorney general of India. So that, there is a fearless participation of the members in the debate
and every member can put forward his thought without any fear or favour.

Some limitations are also present which should be followed in order to claim immunity

Freedom of speech should be in accordance with the constitutional provisions and subject to
rules and procedures of the parliament, stated under Article 118 of the Constitution.

Under Article 121 of the Constitution, the pankajmembers of the parliament are restricted from
discussing the conduct of the judges of the Supreme Court and the High Court. But, even if this
happens, it is the matter of the parliament and the court cannot interfere.

No privilege and immunity can be claimed by the member for anything which is said outside the
proceedings of the house.

Freedom from arrest

The members enjoy freedom from arrest in any civil case 40 days before and after the
adjournment of the house and also when the house is in session. No member can be arrested from
the limits of the parliament without the permission of the house to which he/she belongs so that
there is no hindrance in performing their duties.

If the detention of any members of the parliament is made, the chairman or the speaker should be
informed by the concerned authority, the reason for the arrest. But, a member can be arrested
outside the limits of the house on criminal charges against him under The Preventive Detention
act, The Essential Services Maintenance Act (ESMA), The National Security Act (NSA) or any
such act.

Freedom from appearing as a witness

The members of the parliament enjoy special privileges and are exempted from attending court
as a witness. They are given complete liberty to attend the house and perform their duties
without any interference from the court.

Privileges enjoyed by the members collectively as part of parliament

Right to prohibit the publication of proceedings


10

As stated in Article 105(2) of the Constitution, no person shall be held liable for publishing any
reports, discussions etc. of the house under the authority of the member of the house. For
paramount and national importance, it is essential that the proceedings should be communicated
to the public to aware them about what is going on in the parliament.

But, any partial report of detached part of proceedings or any publication made with malice
intention is disentitled for the protection. Protection is only granted if it reflects the true
proceedings of the house. If any expunged proceedings are published or any misrepresentation or
misreporting is found, it is held to be the breach of the privilege and contempt of the house.

Right to exclude strangers

The members of the house have the power and right to exclude strangers who are not members
of the house from the proceedings. This right is very essential for securing free and fair
pankaj
discussion in the house. If any breach is reported then the punishment in the form of admonition,
reprimand, or imprisonment can be given.

The right to punish members and outsiders for breach of its privileges

The Indian Parliament has the power to punish any person whether strangers or any member of
the house for any breach or contempt of the house. When any breach is committed by the
member of the house, he/she is expelled from the house.

This right has been defined as ‘keystone of parliamentary privilege’ because, without this power,
the house can suffer contempt and breach and is very necessary to safeguard its authority and
discharge its functions. This power has also been upheld by the judiciary in most of the cases.
The house can put in custody any person or member for contempt till the period the house is in
session.

The right to regulate the internal affairs of the house

Each house has a right to regulate its proceedings in the way it deems fit and proper. Each house
has its own jurisdiction over the house and no authority from the other house can interfere in
regulation of its internal proceedings. Under Article 118 of the Constitution, the house have been
empowered to conduct its regulation for proceedings and cannot be challenged in the court of
law on the ground that the house is not in accordance with the rules made under Article 118. The
Supreme Court has also held that this is general provision and the rule is not binding upon the
house. They can deviate or change the rule anytime accordingly.

Punishments prescribed for breach of privileges or contempt of the house

Imprisonment – If the breach committed is of a grave nature the, punishment can be given in
the form of the imprisonment of any member or person.
11

Imposing fine – If in the view of the parliament, the breach or contempt committed is of
economic offence and any pecuniary gain has been made from the breach then, the parliament
can impose fine on the person.

Prosecuting the offenders – The parliament can also prosecute the one committing the breach.

Punishment given to its own members – If any contempt is committed by the members of the
parliament then, he is to be punished by the house itself which could also result in the suspension
of the member from the house.

What constitutes parliamentary breach or contempt of the house?

There is no codification to clearly state that what action constitutes a breach and what
punishment it entails. Although, there are various acts which are treated by the house as the
contempt. It is generally based on the actions which
pankaj tend to obstruct the proceedings of the house
and creates a disturbance for the members. Some of them are briefly discussed.

Giving any misleading statement in the house

The acts which are done solely with the purpose to mislead are considered as the contempt of the
house. If the statement is made by a person who believes the information to be true then, there is
no breach involved. It has to be proved that the statement was made with an intention to mislead
the house.

Disturbance by the outsiders

Any disruption created by shouting slogans or throwing leaflets etc. with the purpose of
disturbing the proceedings of the court is regarded as a major contempt by the house. The person
is imprisoned by the house for a specified period of time or a warning is given depending on the
seriousness of the case.

Any kind of assault on the members

Here, the privilege is available when the member is performing his duties. An assault done by
any person on the member of the parliament in the course of performing his duties is treated as
contempt of the house.

Writings or speeches about the character of the member

any speech published or libel made against the character of the member is regarded as the
contempt of the house. These are regarded to be necessary because it affects the performance and
function of the member by reducing the respect for him.

So, clearly, any attack on the privilege of the members by any means is considered as a breach of
the privilege and the parliament can take action regarding the same.
12

Codification of the parliamentary privileges

Our Indian parliament enjoys supreme powers as being a member of the parliament. There is also
misuse of the privileges given to them because they do not have many restriction on the rights.
They have the power to be the judge of their own proceedings, regulate their proceedings, what
constitutes the breach and what punishment should be given for the breach, are solely decided by
them.

The power vested in them is too wide as compared to the fundamental rights vested in the
citizens. With no codification of the privileges, they have gained an undefined power because
there is no expressed provision to state the limitations on their powers. The privilege from any
civil arrest 40 days before and after the session and during the session results that they are
exempted from arrest for even more than 365 days. No comprehensive law has been till date
enacted by the parliament for the codification of the parliamentary privileges.
pankaj

It is mostly resisted by the members because then it will be subject to the fundamental rights and
would be in the purview of judicial review. Justice M.N. Venkatachaliah heading the
Constitution Review Commission also recommended to define and delimit the privileges for the
free and independent functioning of the legislature. This is based on the apprehension that
codification will involve interference of the court as the matters would be presented in the court
of law. Non-codification of privileges has led to greater powers being enjoyed by the members.
But, now the time has come to codify and define the privileges and actions must be taken so that
there is smooth functioning of the parliament without any conflict.

Judicial Interpretation of Parliamentray Privileges

In ‘Searchlight’ case (I) (M.S.M. Sharma vs. S.K. Sinha, AIR 1959 S.C. 395), Supreme Court
held that the power of judicial review, applicable to ordinary law, could not be invoked to
challenge an order made under Article 194, a Constitutional provision.

In Keshav Singh’s case, the Supreme Court clarified that Article 21 (Right to Life and Personal
Liberty) would be applicable even when Legislatures exercised their powers in respect of their
privilege. However, the position regarding Freedom of Speech being subservient to legislative
privilege was confirmed.

In Raja Ram Pal v Hon’ble Speaker, Lok Sabha and Ors., (2007) 3 SCC 184, The Supreme
Court held that Fundamental Rights under Articles 20 and 21 could prevail over privileges under
Articles 105 and 194. However, no mention was made of rights under Article 19 (1) (a) relating
to Freedom of Speech.

In Algaapuram R Mohanraj v Tamil Nadu Legislative Assembly, WP (C) 455 of 2015 examined
the issue The Supreme Court rejected the violations of the Right to Speech, Right to Expression,
13

Right to Life in the case of breach of privileges, it upheld contention regarding violation of the
Right to Equality was upheld.

Thus the judiciary had been frequently sucked in the case of parliament’s unbridled power in
case of breach of privileges when it comes in conflict with Fundamental Rights.However,
without codification, Searchlight Judgement is still applicable which tilts it in favour of
Parliament.

Arguments against Codification of Parliamentary Privileges

1. It would challenge the Sovereignty of Indian Parliament.

2. It would lead to judicial scrutiny of privileges especially when it comes in conflict with
fundamental rights.
pankaj
3. It would make the evolution of new privileges very difficult.

The members should have the freedom to discharge their functions, including the right to speak
and vote within Parliament, without the fear and favour which requires these parliamentary
privileges.

Argument in favour of Codification of Parliamentary Privileges

The Constitution implicitly provided for a law to codify privileges and adopted the practice in
the House of Commons as a temporary measure. The Constitution says that “In the absence of
any law, such privileges would be the same as those enjoyed by the House of Commons at the
commencement of the Constitution”.However, this provision was amended in 1978 to state that
the privileges would be the same as enjoyed by the Houses of Indian Parliament as on the date of
effect of that amendment.

2. Codification would fix the exact limit of the breach of privileges after which no punishment
can be inflicted.

3. It would define precisely what constitutes and amounts to a breach of privileges.

4. Relief from the court can be easily available if privileges are codified.
14

Unit-11

Qs. Legislative relations between centre and states (Art. 245-254).

Ans. Centre State RelationsThe Constitution of India provides a dual polity with a clear division
of powers between the Union and the States, each being supreme within the sphere allotted to it.
The Indian federation is not the result of an agreement between independent units, and the units
of Indian federation cannot leave the federation. Thus the constitution contains elaborate
provisions to regulate the various dimensions of the relations between the centre and the states.
pankaj
The relations between centre and state are divides as:

1. Legislative relations

2. Administrative relations

3. Financial relations

1. Centre State Legislative Relations

Articles 245 to 255 in Part XI of the Constitution deal with the legislative relations between the
Centre and the State.

Extent of laws made by Parliament and by the Legislatures of State (Art.245)

The Parliament can make laws for the whole or any part of the territory of India. Territory of
India includes the states, UTs and any other area for the time being included in the territory of
India. Whereas, the state legislature can make laws for whole or any part of state.

The Parliament can alone make ‘extra territorial legislation’ thus the laws of the Parliament are
applicable to the Indian citizens and their property in any part of the world.

Subject-matter of laws made by Parliament and by the Legislation of States (Art. 246)

The Constitution divides legislative authority between the Union and the States in three lists- the
Union List, the State List and the Concurrent List. The Union list consists of 99 items. The
Union Parliament has exclusive authority to frame laws on subjects enumerated in the list. These
include foreign affairs, defence, armed forces, communications, posts and telegraph, foreign
trade etc.
15

The State list consists of 61 subjects on which ordinarily the States alone can make laws. These
include public order, police, administration of justice, prison, local governments, agriculture etc.

The Concurrent list comprises of 52 items including criminal and civil procedure, marriage and
divorce, economic and special planning trade unions, electricity, newspapers, books, education,
population control and family planning etc. Both the Parliament and the State legislatures can
make laws on subjects given in the Concurrent list, but the Centre has a prior and supreme claim
to legislate on current subjects. In case of conflict between the law of the State and Union law on
a subject in the Concurrent list, the law of the Parliament prevails.

Residuary powers of legislation (Art. 248)

The constitution also vests the residuary powers (subjects not enumerated in any of the three
Lists) with the Union Parliament. The residuary powers have been granted to the Union contrary
pankaj where the residuary powers are given to the
to the convention in other federations of the world,
States. However, in case of any conflict, whether a particular matter falls under the residuary
power or not is to be decided by the court.

Parliament’s Power to Legislate on State List (Art. 249,250,251,252,253)

Though under ordinary circumstances the Central Government does not possess power to
legislate on subjects enumerated in the State List, but under certain special conditions the Union
Parliament can make laws even on these subjects.

a) In the National Interest (Art.249)

If the Rajya Sabha declares by a resolution supported by not less than 2/3 of its members present
and voting, that it is necessary or expedient in the national interest that the Parliament should
make laws with respect to any matter enumerated in the State List (Art.249). After such a
resolution is passed, Parliament can make laws for the whole or any part of the territory of India.
Such a resolution remains in force for a period of 1 year and can be further extended by one year
by means of a subsequent resolution.

b) Under Proclamation of National Emergency (Art.250)

Parliament can legislate on the subjects mentioned in the State List when the Proclamation of
National Emergency is in operation. However, the laws made by the Parliament under this
provision shall cease to have effect on the expiration of a period of six months after the
Proclamation has ceased to operate, except as respects things done or omitted to be done before
the expiry of the said period.

c) By Agreement between States (Art. 252)

The Parliament can also legislate on a State subject if the legislatures of two or more states
resolve that it is lawful of Parliament to make laws with respect to any matter enumerated in the
16

State List relating to those State. Thereafter, any act passed by the Parliament shall apply to such
states and to any other state which passes such a resolution. The Parliament also reserves the
right to amend or repeal any such act.

d) To Implement Treaties (Art. 253)

The Parliament can make law for the whole or any part of the territory of India for implementing
any treaty, international agreement or convention with any other country or countries or any
decision made at any international conference, association or other body. Any law passed by the
Parliament for this purpose cannot be invalidated on the ground that it relates to the subject
mentioned in the State list.

Inconsistency of a law (Art. 254)

Article 254 Article 254 says that if there is anypankaj


inconsistency of a law made by the Legislature of
a State with any provision of a law made by Parliament which Parliament is competent to enact,
or on a matter of Concurrent List, then, the law made by Parliament, whether passed before or
after the law made by the Legislature of such State, or, as the case may be, the existing law, shall
prevail.

Doctrine of Colourable Legislation

This is applied when the legislature enacting the law has transgressed its power as is mentioned
in the Constitution. The expression “colourable legislation” simply means what cannot be done
directly, cannot be done indirectly too. It is the substance that is material and not the outward
appearance. Hence there are certain situations when it seems that it is within the power of the
legislature enacting the law but actually it is transgressing. This is when this doctrine comes into
the picture.

It was applied by the Supreme Court of India in the case State of Bihar vs Kameshwar Singh and
it was held that the Bihar Land Reforms Act was invalid.

Doctrine of Pith and Substance

This doctrine comes into picture when there is a conflict between the different subjects in
different lists. There is an interpretation of List 1 and List 2 of the Constitution of India. There
can be a situation when a subject of one list touche the subject of another List. Hence this
doctrine is applied then. Pith and Substance means the true nature of law.

The real subject matter is challenged and not its incidental effect on another field. The doctrine
has been applied in India also to provide a degree of flexibility in the otherwise rigid scheme of
distribution of powers. The reason for the adoption of this doctrine is that if every legislation
were to be declared invalid on the grounds that it encroached powers, the powers of the
17

legislature would be drastically circumscribed. It was applied by the Supreme Court in the
case State of Bombay Vs F.N Balasar.

2. Centre State Administrative Relations

The administrative jurisdiction of the Union and the State Governments extends to the subjects in
the Union list and State list respectively. The Constitution thus defines the clauses that deal with
the administrative relations between Centre and States. Centre State Relations During Normal
Ties

1. Executive Powers of State be exercised in compliance with Union Laws: Article 256 lays
down that the executive power of every State shall be so exercised as to ensure compliance with
the laws made by Parliament and any existing laws which apply in that State, and the executive
power of the Union shall extend to the giving of such directions to a state as may appear to the
pankaj
Government of India to be necessary for that purpose.

2. Executive Powers of State not to interfere with Executive Power of Union: Article 257 of
the Constitution provides that the executive power of every state shall be so exercised as not to
impede or prejudice the exercise of the executive power of the Union, and the executive power
of the Union shall extend to giving of such directions to a state as may appear to the Government
of India to be necessary for that purpose. In short, the Union Government can issue directions to
the state Government even with regard to the subjects enumerated in the state list.

3. Maintain means of communication of National or Military importance: The Union


Government can give directions to the state with regard to construction and maintenance of the
means of communication declared to be of national or military importance.

4. Protection of the Railways: Union can issue State Governments necessary directions
regarding the measures to be taken for the protection of the railways within the jurisdiction of the
State. It may be noted that the expenses incurred by the State Governments for the discharge of
these functions have to be reimbursed by the Union Government.

5. To ensure welfare of Scheduled Tribes in the States: Union can direct the State Governments
to ensure execution of schemes essential for the welfare of the Scheduled Tribes in the States.

6. To secure instruction in the mother-tongue at the primary stage of education: Union can direct
the State Governments to secure the provision of adequate facilities for instruction in the mother-
tongue at the primary stage of education to children belonging to linguistic minority groups.

7. To ensure development of the Hindi language: Union can direct the State Governments to
ensure the development of the Hindi language.
18

8. To ensure government of a State is carried on in accordance with the provision of the


Constitution: Union can direct the State Governments to ensure that the government of a State is
carried on in accordance with the provision of the Constitution. If any State failed to comply
with any directions given by the Union in exercise of its executive power, then President may
hold that, a situation has arisen in which the Government of the State cannot be carried on in
accordance with the provisions of the Constitution. Thus he may proclaim President’s Rule in
that State.

9. Delegation of Union’s function to State: The President of India can entrust to the officers of
the State certain functions of the Union Government. However, before doing so the President has
to take the consent of the state Government. But the Parliament can enact law authorizing the
Central Government to delegate its function to the State Governments or its officers irrespective
of the consent of such State Government. On the other hand, a State may confer administrative
functions upon the Union, with the consent of the Union only.
pankaj

10. Appointment of High Dignitaries: Union has major say in appointment and removal of
Governor and appointment of Judges of High Court and Members of State Public Service
Commission.

11. All India Services: The presence of the All India Services - the Indian Administrative
Services, Indian police Services - further accords a predominant position to the Union
Government. The members of these services are recruited and appointment by the Union Public
Service Commission. The members of these services are posted on key posts in the states, but
remain loyal to the Union Government.

12. Union to adjudicate Inter-State River Water Dispute: The Parliament has been vested with
power to adjudicate any dispute or complaint with respect to the use, distribution or control of
the waters of, or in any inter-state river or river-valley. In this regard, the Parliament also
reserves the right to exclude such disputes from the jurisdiction of the Supreme Court or other
Courts.

3. Centre State Financial Relations:

Indian Constitution has made elaborate provisions, relating to the distribution of the taxes as well
as non-tax revenues and the power of borrowing, supplemented by provisions for grants-in-aid
by the Union to the States. Article 268 to 293 deals with the provisions of financial relations
between Centre and States.

The Constitution divides the taxing powers between the Centre and the states as follows:

The Parliament has exclusive power to levy taxes on subjects enumerated in the Union List, the
state legislature has exclusive power to levy taxes on subjects enumerated in the State List, both
19

can levy taxes on the subjects enumerated in Concurrent List whereas residuary power of
taxation lies with Parliament only.

Distribution of the tax-revenue

1. Duties Levied by the Union but Collected and Appropriated by the States: Stamp duties on
bills of Exchange, etc., and Excise duties on medical and toilet preparations containing alcohol.
These taxes don’t form the part of the Consolidated Fund of India, but are assigned to that state
only.

2. Service Tax are Levied by the Centre but Collected and Appropriated by the Centre and the
States.

3. Taxes Levied as Well as Collected by the Union, but Assigned to the States: These include
taxes on the sale and purchase of goods in the course of inter-state trade or commerce or the
pankaj
taxes on the consignment of goods in the course of inter-state trade or commerce.

4. Taxes Levied and Collected by the Union and Distributed between Union and the States:
Certain taxes shall be levied as well as collected by the Union, but their proceeds shall be divided
between the Union and the States in a certain proportion, in order to effect on equitable division
of the financial resources. This category includes all taxes referred in Union List except the
duties and taxes referred to in Article 268, 268-A and 269; surcharge on taxes and duties
mentioned in Article 271 or any Cess levied for specific purposes.

5. Surcharge on certain duties and taxes for purposes of the Union: Parliament may at any time
increase any of the duties or taxes referred in those articles by a surcharge for purposes of the
Union and the whole proceeds of any such surcharge shall form part the Consolidated Fund of
India.

Grants-in-Aid
Besides sharing of taxes between the Center and the States, the Constitution provides for Grants-
in-aid to the States from the Central resources. There are two types of grants:-

1. Statutory Grants: These grants are given by the Parliament out of the Consolidated Fund of
India to such States which are in need of assistance. Different States may be granted different
sums. Specific grants are also given to promote the welfare of scheduled tribes in a state or to
raise the level of administration of the Scheduled areas therein (Art.275).

2. Discretionary Grants: Center provides certain grants to the states on the recommendations of
the Planning Commission which are at the discretion of the Union Government. These are given
to help the state financially to fulfill plan targets (Art.282).

Effects of Emergency on Center-State Financial Relations:-


20

1. During National Emergency: The President by order can direct that all provisions regarding
division of taxes between Union and States and grants-in-aids remain suspended. However, such
suspension shall not go beyond the expiration of the financial year in which the Proclamation
ceases to operate.

2. During Financial Emergency: Union can give directions to the States:-


1. To observe such canons of financial propriety as specified in the direction.
2. To reduce the salaries and allowances of all people serving in connection with the affairs of
the State, including High Courts judges.

3. To reserve for the consideration of the President all money and financial Bills, after they are
passed by the Legislature of the State.

Finance Commission
pankaj
Although the Constitution has made an effort to allocate every possible source of revenue either
to the Union or the States, but this allocation is quite broad based. For the purpose of allocation
of certain sources of revenue, between the Union and the State Governments, the Constitution
provides for the establishment of a Finance Commission under Article 280. According to the
Constitution, the President of India is authorized to set up a Finance Commission every five
years to make recommendation regarding distribution of financial resources between the Union
and the States.

Constitution

Finance Commission is to be constituted by the President every 5 years. The Chairman must be a
person having ‘experience in public affairs’. Other four members must be appointed from
amongst the following:-

1. A High Court Judge or one qualified to be appointed as High Court Judge;

2. A person having knowledge of the finances and accounts of the Government;

3. A person having work experience in financial matters and administration;

4. A person having special knowledge of economics.

Functions

The Finance Commission recommends to the President as to:-

1. The distribution between the Union and the States of the net proceeds of taxes to be divided
between them and the allocation between the States of respective shares of such proceeds;

2. The principles which should govern the grants-in-aid of the revenue of the States out of the
Consolidated Fund of India;
21

3. The measures needed to augment the Consolidated Fund of a State to supplement the
resources of the Panchayats and Municipalities in the State;

4. Any other matter referred to the Commission by the President in the interest of sound finance

Conclusion:

In India, the Centre-States relations constitute the core elements of the federalism. The Central
Government and State Government cooperate for the well-being and safety of the citizens of
India. The work together in the field of environmental protection, terror control, family control
and socio-economic planning. The Indian constitution aim at reconciling the national unity while
giving the power to maintain state to the State governments. It is true that the union has been
assigned larger powers than the state governments, but this is a question of degree and not
quality, since all the essential features of a federation are present in the Indian constitution. It is
often defined to be quasi-federal in nature. Thus,pankaj
it can be safely said that Indian Constitution is
primarily federal in nature even though it has unique features that enable it to assume unitary
features upon the time of need. Federal but its spirit is unitary.

Qs. Special status of J&K-Art. 370

Article 370 in the part XXI of the constitution of India provides a special status to the state of
Jammu and Kashmir. The special status provided the state with considerable autonomy and most
of the decision was taken by center and in consent with the wishes of the state government,
excluding external affairs, defence, communications and ancillary matters.

What is Article 370?

Article 370 of the Indian Constitution confers special autonomous status to Jammu & Kashmir. It
is a 'temporary provision' under Part XXI of the Constitution of India, which deals with
"Temporary, Transitional and Special Provisions." The state has different provisions than all
other states, according to the Constitution. For example, till 1965, J&K had a prime minister in
place of the chief minister.

History of Article 370

The article provision was drafted by Sheikh Abdullah, since he didn’t desire temporary
provisions for Article 370. Sheikh Mohammad Abdullah, the state's 'Prime Minister' and a
prominent leader of the Muslims in the Valley, found the inclusion of Article 370 in the
'Temporary and Transitional Provisions' of the Constitution's Part XXI and disagreed. He wanted
'iron clad guarantees of autonomy'. Since he suspected that the state's special status might be lost,
and began to advocate freedom from India. This resulted in the dismissing his government in
1953, and place him under preventive detention.
22

After five decades, the Supreme Court of India set aside a judgement of the High Court of
Jammu and Kashmir which stated that Jammu and Kashmir had "absolute sovereign power" on
account of Article 370, in December 2016. The Supreme Court held that the state of Jammu and
Kashmir has "no vestige" of sovereignty outside the Constitution of India and its own
Constitution is subordinate to the Indian Constitution. The Court upheld the applicability of
SARFAESI Act to Jammu and Kashmir as it was under the Union list of subjects for which the
Indian Parliament is empowered to enact laws for the whole of India, including Jammu and
Kashmir.

Main Features of Article 370

Though, envisaged to be of very temporary arrangement, it went on to define the special


treatment to the state of Jammu and Kashmir as elaborated below:
pankaj
1. Under the agreement, the state surrendered defence, communications and external affairs.

2. State was also provided with autonomy to draft its own constitution through its separate
constituent assembly.

3. Article 370 was incorporated in Indian constitution temporarily to accommodate above


features.

4. The state of J&K was specified in the category of Part B states in the original constitution
(1950).

5. Laws on Union and concurrent list will be made by centre only with the consent of state of
J&K 1. Its name, area or boundary cannot be changed without the consent of state legislature.

6. Part VI of Indian constitution dealing with state government is not applicable.

7. Residuary power belongs to state, except in prevention of activities involving terrorist acts,
questioning and disrupting the sovereignty and territorial integrity of India and causing insult to
the national flag, national anthem and the constitution of India.

8. Fundamental right to property is still granted in the state.

9. Special rights are granted to permanent residents of state with regard to public employment,
acquisition of immovable property, settlement and government scholarships.

10. Directive Principles of state Policies and Fundamental duties are not applicable.

11. National emergency on the ground of internal disturbance will have no effect except with the
concurrence of state government.

12. Financial emergency cannot be imposed.


23

13. High Court of Jammu and Kashmir cannot issue writs for other than Fundamental rights.

14. The denial of citizenship rights of migrants to Pakistan is not applicable.

15. Fifth schedule and sixth schedule of Indian Constitution are not applicable.

16. Official language provisions are applicable only in so far as they relate to the official
language of the Union.

17. An amendment made to the Constitution of India does not apply automatically to the state
unless extended by Presidential Order.

18. President rule can be applied only on the ground of failure of constitutional machinery of
state constitution and not of Indian constitution.

19. Consent of State legislature is also imperative in case of international treaties or agreements.
pankaj

Thus, the relation between Jammu and Kashmir and India must be harmonized at the earliest in
order for India to attain glory. It cannot bear the brunt of boiling Kashmir if it wants to develop
and prosper in the contemporary world. The panacea lies in addressing the aspirations of all the
stake holders on priority basis without compromising the integrity and security of India. India
will develop if all its organs and its geography develop.
24

Unit-III

Emergency provision (Art. 352-360)

The Emergency provisions for India are mentioned in the Part XVIII of the constitution from
Articles 352 to 360. Reason for Emergency Provisions in India: These Emergency provisions are
included in the constitution to preserve the sovereignty, Unity, Integrity & Security of the
country. There are 3 emergencies mentioned inpankaj
the constitution. Types Of Emergencies in India:

1. National Emergency

2. State Emergency

3. Financial Emergency

1. National Emergency

If the president is satisfied that a grave emergency exists whereby the security of India or of any
part of the territory thereof is threatened, whether by war or external aggression or armed
rebellion, he may, by proclamation, made a declaration to that effect in respect of the whole of
India or of such part of the territory thereof as may be specified in the proclamation.

Grounds for proclamation of emergency: – ‘National Emergency’ is imposed due to war,


external aggression or armed rebellion i.e. when there is a threat to the security of the country or
any territory thereof. The president shall not issue a proclamation unless a decision of the union
cabinet that such a proclamation may be issued (or varied), has been communicated to him in
writing. Such a proclamation may be made even before the actual war, external aggression or
rebellion if the president is satisfied that there is imminent danger thereof.

The provisions of article 352 were made more stringent by the constitution 44th amendment act,
1978. To prevent the misuse of emergency provisions, the words ‘armed rebellion’ were
substituted for ‘internal disturbance’. The expression “internal disturbance” was found to be too
vague and wide; it may cover a minor disturbance of law and order or even a political agitation.
25

it gave wide discretion to the executive to declare emergency even on flimsy grounds. for
instance, in 1975, the emergency was declared on the ground of internal disturbance by the then
prime minister Indira Gandhi because the opposition parties had given a call to launch a
movement with a view to compelling her to resign from her post as her election to the lok sabha
was declared void by the Allahabad high court.

Thus, after the 44th amendment, ‘internal disturbance not amounting to armed rebellion’ would
not be a ground for the issue of a proclamation of emergency. This change has restricted the
scope of what may be known as ‘internal emergency.’

It may also be noted that the president shall declare emergency only on the ‘written advice’ of
pankaj
the cabinet and not merely on the advice of prime minister as was done by the Prime Minister
Indira Gandhi in 1975. She had advised the president to proclaim emergency without consulting
her cabinet. The cabinet was simply informed about the proclamation of emergency. Thus, cl. (3)
was added to article 352 by the 44th amendment to prevent the recurrence of such a situation in
future.

There have been three proclamations of national emergency in India – in 1962 at the time of
Chinese aggression, in 1971 in the wake of war with Pakistan and in June 1975 on grounds of
internal disturbance.

Approval and Duration of proclamation: – Every proclamation is required to be laid before


each house of the parliament, and is to cease to operate at the expiration of 1 month (before 44th
amendment, it was two months, without the approval of both houses of parliament) from the date
of its issue unless in the meantime it has been approved by resolutions of both the houses.
However, once approved by parliament, the proclamation may continue for 6 months at a
time unless revoked by the president earlier. Resolutions approving the proclamation or its
continuance have to be passed by both house of parliament by a majority of the total membership
and not less than 2/3rd of those present and voting.

Provided that if and so often as a resolution approving the continuance in force of such a
proclamation is passed by both houses of parliament the proclamation shall, unless revoked,
continue in force for a further period of six months from the date on which it would otherwise
have ceased to operate under this clause.
26

Revocation of proclamation: – Prior to 44th amendment, a proclamation of emergency once


approved by both houses of parliament could be revoked by the president by making a fresh
proclamation. Thus, the executive was the sole judge to decide as to when the proclamation
should be revoked. In other words, once approved by parliament, emergency could remain in
force indefinitely i.e. so long as the executive wanted it to continue. There was no provision for
periodical review by parliament every six months.

Now, after the 44th amendment, firstly, a proclamation may be revoked by the president by
making a subsequent proclamation. Secondly, the 44th amendment requires a periodical review,
every six months, of the proclamation by both houses of parliament. thirdly, the president shall
revoke the proclamation of emergency, if the lok sabha passes a resolution disapproving the
pankaj
proclamation (or varying it) or its continuance in force; for this, a notice in writing signed by not
less than one-tenth of the total number of members of the lok sabha is needed.

Judicial review of proclamation: – A view was prevalent that the question whether emergency
exists is essentially a political question entrusted by the constitution to the union executive and
therefore not justiciable before the court.

However, in Minerva Mills Ltd. verses UOI, it was held that there is no bar to judicial review
of the validity of a proclamation of emergency issued by the president under article. 352. Merely
because a question has a political complexion, it is no ground why the court should shrink from
performing its duty under the constitution if it raises an issue of constitutional determination. The
court’s power, however, is limited only to examining whether the limitations conferred by the
constitution have been observed or not. the court cannot go into question of adequacy of the facts
and circumstances on which the president’s satisfaction is based unless it can be shown that there
is no satisfaction of the president at all (in that case the exercise of the power would be
constitutionally invalid). Where at all, the satisfaction is absurd or perverse or mala fide or based
on wholly extraneous and irrelevant grounds, it would be no satisfaction at all and it would be
liable to be challenged before a court of law.

Consequences of proclamation of emergency

(1) Effect of proclamation of emergency: – Though the state legislature and government are
not suspended, the executive, legislative and financial powers rest in the Centre. Notwithstanding
anything in this constitution, the executive power of the union shall extend to the giving of
27

directions to any state as to the manner in which the executive power thereof is to be exercised.
The union parliament acquires the power to legislate on any subject included in the state list
(article 353). But such emergency legislation ceases to have any effect at the expiry of 6 months
after the proclamation ceases to operate. also, it may be noted that the executive power of the
union to give directions and the power to make laws shall also extend to any state other than the
state where emergency is in force, if the security of India or any part of the territory is threatened
by activities in or in relation to that part of the territory of India in which the proclamation of
emergency is in operation.

Again, it may be noted that the law-making power of the state is not suspended during the
emergency. The state can make law but it is subject to the overriding power of the union
pankaj
parliament. Article 354 provides for acquisition of powers by union executive relating to
distribution of revenues.

While a proclamation of emergency is in operation, life of parliament can be extended by law for
a period not exceeding one year at a time and not extending in any case beyond a period of six
months after the proclamation has ceased to operate [article 83(2)].

(2) Effect on fundamental rights (articles 358-359)

As soon as the emergency is proclaimed on the ground of war or external aggression, all the
freedoms guaranteed by article 19 are automatically suspended. art. 358 make it clear that in the
case of proclamation under article. 352, article. 19 shall not restrict the power of the state to
make any law or to take any executive action abridging or taking away the fundamental rights
guaranteed by art. 19. But, any law so made shall, to the extent of the incompetency, cease to
have effect as soon as the proclamation ceases to operate. However, things done or omitted to be
done during emergency cannot be challenged even after the emergency was over on the ground
of the concerned emergency law had violated article 19.

It may be noted that 44th amendment excluded the ground of ‘armed rebellion’; thus, if
emergency is declared on this ground, freedoms guaranteed by art. 19 cannot be suspended.
Further, the amendment made it clear that article 358 will only protect ’emergency laws’ from
being challenged in a court of law and no other laws which are not related to the emergency.
Prior to this, the validity of even other laws, which were not related to emergency, could not be
challenged under article 358.
28

article 359(1) lays down that where a proclamation of emergency is in operation, the president
may by order declare that the right to move any court for the enforcement of such of the rights
conferred by part 111 (except articles 20 and 21) as may be mentioned in the order and all
proceedings pending in any court for the enforcement of the rights so mentioned shall remain
suspended for the period during which the proclamation is in force or for such shorter period as
may be specified in the order. Thus, the president has the power to suspend the right to move
courts for the enforcement of any of the fundamental rights (except articles. 20 and 21) for a
period not exceeding the proclamation of emergency.

The main difference between article 358 and article 359 is that fundamental rights are
automatically suspended during emergency (article. 358), while under article. 359 it is president
pankaj
who gives the order. However, every order made under clause (1) shall, as soon may be after it is
made, be laid before each house of parliament. Further, under article. 359, the fundamental rights
are not suspended, it is the right to seek remedy that is suspended.

The proclamation of emergency does not invalidate a law which was valid before the
proclamation.

Duty of union to protect states (article 355)

Article 355 lays down that “it shall be the duty of the union to protect every state against external
aggression and internal disturbance and to ensure that the government of every state is carried on
in accordance with the provisions of this constitution.”

Such provisions are also found in other federal constitutions i.e. USA and Australia. But in US
and Australia the center acts only when the request is made by states, while there is no such pre-
condition under article. 355. The center can thus interfere even without the state’s request.
Further, it is this duty in performance of which the Centre takes over the government of state
under article 356 in case of failure of the constitutional machinery in the state. in other
federations, the centre cannot do so.

Article 355 may be invoked by the centre to interfere in the affairs of the state under certain
situations like state’s failure to provide adequate protection to the central government agencies in
the state, the destruction of the properties of central government by agitators, etc. on several
occasions, the central government has appointed commissions of inquiry under commission of
29

inquiry act, 1952, to go into charges of omission or commission, bordering on corruption, against
the state chief ministers, ex-chief minister, ex-ministers.

President’s rule in states (‘state emergency’) (article 356)

‘‘Failure of constitutional machinery in state” – if the president on governor’s report, or


otherwise is satisfied that a situation has arisen in which government of a state can’t be carried
on in accordance with the constitutional provisions, he may issue a proclamation to that effect
[cl. (1)].

The following consequences ensue on the issuance of a proclamation under article 356(1):-

(1) President may assume to himself all or any of the functions of the state government, or
powers vested in governor, or anybody or authority in the state other than state legislature.
pankaj

(2) President may declare that powers of legislature of state shall be exercisable by parliament.

(3) The president may make such incidental and consequential provisions as may appear to him
to be necessary or desirable for giving effect to the object of proclamation.

The president cannot, however, assume to himself, any of the powers vested in high court, or to
suspend any constitutional provision relating to it [proviso, article. 356(1)].

The proclamation issued under article. 356(1) may be revoked or varied by the president on a
subsequent proclamation [article. 356(2)].

it is to be noted that under article. 356 the president acts on a report of the governor or on
information received otherwise. This means that the president can act even without the
governor’s report. This is justified in view of the obligation of the centre imposed by article
355 to ensure that the state government is carried on in accordance with the constitutional
provisions. In view of the centre’s ultimate responsibility to protect the constitutional machinery
of the states, the framers thought it proper not to restrict the centre’s action merely on the
governor’s report. The governor might not sometimes make a report.

Article 365 also provides that where any state failed to comply with or to give effect to union
directives; it shall be lawful for the president to hold that such a situation has arisen in which the
state government cannot be carried on in accordance with the constitutional provisions.

Duration of proclamation under article 356

Article 356(3) provides that a proclamation relating to state emergency shall be laid before each
house of parliament and unless both houses approve it by a resolution, it shall cease to have
effect (except the one which revokes the earlier one) at the expiration of two months, unless in
30

the meantime, it has been approved by resolutions of both houses of parliament. If approved, it
will be valid for six months.

Article 356(4) provides that the duration of proclamation can be extended by 6 months each time
by both houses of parliament passing resolutions approving its continuance. the proviso to clause
(4) lays down that the maximum period for which a proclamation can remain in operation is 3
years from the date it is issued under article 356(1). Thereafter, the president’s rule must come to
an end and the normal constitutional machinery to be restored in the state.

Article 356(5) [inserted by 44th amendment], however, lays down that beyond 1 year a
proclamation can be continued only if emergency under article. 352 were in operation in the
whole of India or in the whole or any part of the concerned state, and the election commission
certifies that it is not possible to hold elections to the state legislature. This provision has put a
restraint on the power of parliament to extend a proclamation; it can be extended beyond one
pankaj
year only if special circumstances exist.

It may be noted that in Punjab, the president’s rule was imposed for 5 years but for this the
constitution was amended several times.

Revocation of proclamation under article 356

A proclamation issued under article 356(1) expires:

(1) After two months of its making, if it is not presented for approval before both houses of
parliament.

(2) Even before two months, if the proclamation on presentation to the houses of parliament, fail
to get approval from any house.

(3) after 6 months from the date of proclamation, in case no further resolution is passed by the
houses of parliament after the passage of the initial resolution approving the said proclamation.

(4) After the expiry of 6 months from the passage of the last resolution of approval passed by
parliament subject to an overall maximum limit of 3 years from the date of proclamation.
Continuance of the proclamation beyond one year is subject to special circumstances.

(5) The date on which the president issues a proclamation of revocation.

Proper grounds for application of article 356

(1) Hung Assembly Scenario – Where, after a general election, no party is able to secure a
working majority in the legislative assembly. Or, where the party having a majority declines to
form a ministry and the governor’s attempt to find a coalition ministry able to command a
majority have failed. Or, where a ministry having resigned & the governor finds it impossible to
form an alternative government.
31

It may be noted that when a new state is created as a result of territorial reorganization or
upgrading of a union territory and there is no legislature for such state until election is held
therefore, resort may be had to article 356 as a “stop-gap” arrangement.

(2) Corruption, maladministration, etc. – president’s rule may be imposed when there is gross
mismanagement of the affairs of a state government, or abuse of its power, or corruption on the
part of the state government.

(3) Acting contrary to the constitution of India – A subversion of the constitution by the state
government while professing to work under the constitution or creating disunity or disaffection
among the people to disintegrate the democratic social fabric, or to subvert its ‘basic features’
such as federation, secularism or democracy. Thus, the president’s rule may be imposed when a
political party seeks to subvert the principles of responsible government and sets up a party
dictatorship.
pankaj

(4) Acting contrary to the union directives – where a state government fails to comply with
the directions issued by the union under the article 257(2)-(3); 353a; 360(3); 339(2), even after
warning.

(5) Failure to meet an extraordinary situation – example an outbreak of unprecedented


violence, a great natural calamity such as a severe earthquake, a flood, or a large epidemic, etc.
failure to meet such situation amounts to an abdication of its governmental power by the state
government.

(6) Security concerns – a danger to national integration or security of the state (calling for an
application of articles. 352 or 355) or aiding or abetting national disintegration or a claim for
independent sovereign status.

Impact

Distinction between article 352 and article 356

(1) Article 352 (‘national or internal emergency’) restricts central government’s intervention to a
situation of war, external aggression, or armed rebellion. Article 356 (‘state emergency’) applies
to a situation of failure of constitutional machinery in a state.

That means under article 352, the relationship of all states with the centre undergoes a change.
But, under article 356, the relationship of only the concerned state with the centre is affected.
32

(2) It may be noted that in comparison to article 352, under article. 356, the state legislature
remains suspended and dissolved. Laws for the state are made by parliament and the governor
administers the state on behalf of the president.

in case of national emergency, the state governments and legislatures continue to function
normally and exercise the powers assigned to them under the constitution. all that happens under
article. 352 is that the centre gets concurrent powers of legislation in state matters and thus it can
make the states follow a uniform all-India policy.

(3) While article. 352 affects fundamental rights, fundamental rights remain unaffected under
article. 356.
pankaj

(4) A proclamation under article 352 has to be approved by parliament within a month and
thereafter every six months. But, there is no maximum duration prescribed for the operation of
such a proclamation. Thus, it can be continued indefinitely.

The proclamation under article 356 is to be approved by parliament within two months, and
thereafter every six months, and the maximum period for which it can remain in force is three
years.

(5) Though the scope and purpose of articles 352 and 356 are very different, there might arise a
situation when article 356 may have to be invoked to effectuate article 352 examples when a
state government does not cooperate with the centre in defense matters, or in quelling internal
disturbances, or when it encourages the same.

Financial Emergency (Article 360)

The president is authorized by article 360 to declare by a proclamation financial emergency if he


is satisfied that the financial stability or credit of India or any part of its territory is threatened. It
has to be laid before both houses of parliament and ceases to operate at the expiration of two
months unless meanwhile approved by the resolutions of the two houses. Once approved by
parliament, unlike proclamations under article 352, it may continue indefinitely until revoked or
varied.
33

During the operation of financial emergency, the executive authority of the union extends to the
giving of directions to any state to observe certain specified canons or financial propriety and
such other direction that the president may find necessary or adequate. These directions may
include reduction of salaries or allowances of all those serving a state and reserving for the
president’s consideration all money bills and other bills under art. 207 after these are passed by
the state legislatures. The president may also direct reduction in salaries, etc. of all those serving
in connection with the affairs of union including judges of the Supreme Court and the high
courts. Thus far, there has been no occasion for the promulgation of financial emergency in
India.

Qs. Amendment of Constitution – Art 368


pankaj

Ans. The Constitution Of India declares India into a ‘Sovereign, Socialist, Secular, Democratic,
Republic’ country. The Constitution of India was passed by the constituent assembly on 26
November 1949. The Constitution of India is considered as the longest written Constitution has
395 Articles and 12 Schedules. A Constitution should be dynamic in nature and should able to
adapt itself to the changing needs of society. As due to the sudden change in the society, the
Constitution and the pattern of government will require a major change. Article 368 of the Indian
Constitution provides the procedure of Amendment. Indian Constitution is neither rigid nor
flexible because, under Article 368, the Constitution can be amended by a simple majority or by
the special majority and by the majority of not less than 2/3 members of each house.

There are two types of Amendment procedures

Rigid

Under this procedure, it is difficult to amend the Constitution. This procedure is used by the
U.S., Australia, Canada, and Switzerland.

Flexible

Under this procedure, it is easy to amend the Constitution. The Amendment can be done by
passing normal legislation.

Indian Constitution is both rigid as well as flexible i.e. it is difficult to amend but practically
flexible. As per Article 368 of the Indian Constitution, an Amendment can be introduced in
either of the houses, later it can be passed by a special majority or by a simple majority. Later if
the bill is passed by the majority it will be sent to the president for his assent.
34

In 69 years of the Constitution, 103 Amendments are already done. The 42nd Amendment is
considered as the terms socialist, secular, integrity was inserted through it. The First Amendment
was done in the year 1950, itself.

Necessity of Amendment

The time is not static, it goes on changing. It is necessary to make changes in the Constitution.
The social, economic, political condition of the people goes on changing. If the changes were not
done in the Constitution we would be unable to encounter the future difficulties and it will
become a hurdle in the path of development.

Kinds of Amendment of the Constitution

The Constitution of India provides for the amendment through Amendment Acts in a formal
manner. For the purpose of amendment, the variouspankaj Articles of the Constitution are divided into
three categories. The first category is out of the purview of Art.368 whereas the other two are a
part and parcel of the said Article. It enables Indian Parliament to amend any provisions of
Constitution without disturbing its Basic Structure. The amendments described in Art.368 are of
three types

1) Amendment by simple majority vote of the Parliament

2) Amendment by special majority vote of the Parliament

3) Amendment by special majority vote with half number of States' legislative Assemblies
ratification.

The various categories of amendment to the Constitution can be summarized as follows:

1. Amendment by simple majority vote of the Parliament

As the name suggests, an Article can be amended in the same way by the Parliament as an
ordinary law is passed which requires simple majority. Such amendment may be moved

At the instance of Union Government

At the instance of a State Government

1 At the instance of Union Government: The following Constitutional provisions can be


amended by a simple majority of the Parliament at the instance of Union Government.

a) Admission of new State under Art. 2 along with Schedule I and Schedule IV

b) Provisions relating to citizenship of India


35

c) Provisions relating to exercise of executive power by a State of its officers in respect of


matters over which Parliament has power to make laws

d) Provisions relating to salaries and allowances of Ministers

e) Provisions relating to salaries and allowances of Speaker and Deputy Speaker, Chairman and
Deputy Chairman

f) Provisions relating to salaries and allowances of members of Parliament

g) Provisions relating to number of Judges in the Supreme Court

h) Provisions relating to privileges, rights and allowances of Supreme Court

i) Provisions relating to appeal to Supreme Court65 j) Provisions relating to review the judgment
of the Supreme Court pankaj

k) Provisions relating to salaries and allowances of Comptroller and Auditor General

l) Provisions relating to composition of the Legislative Councils in States

m) Provisions relating to salaries and allowances of Judges of High Court

n) Provisions relating to continuance of English language

o) Provisions relating to language to be used in Supreme and High Court

p) Provisions relating to creation of Legislative and Council of Ministers for Union Territories

q) Provisions relating to administration and control of the Schedule areas and Schedule Tribes

r) Provisions relating to administration and control of tribal areas

2 At the instance Government of a State

This can be considered under the two categories:

a) Some of the provisions of Constitution can be amended at the instance of a State. Provisions
relating to upper house in the States75fell under this category.

b) Some of the provisions of the Constitution can be amended with the consultation of States.
Provisions relating to formulation of a new State and alterations of areas, boundaries or names of
any State fall under this category.76These Articles are specifically excluded from the purview of
the procedure prescribed under Article 368.

2 Amendment by special majority vote of the Parliament Articles which can be amended by
special majority are laid down in Art.368. All amendments, except those referred to above come
36

within this category and must be affected by a majority of total membership of each House of the
Parliament as well as 2/3rd of the members present and voting.

3 Amendment by Special Majority Vote with half number of States Legislative Assemblies
Ratification Amendment to certain Articles requires special majority as well as ratification by
States. Proviso to Art.368 lays down the said rule. Ratification by States means that there has to
be a resolution to that effect by one-half of the State legislatures. These articles include.

a) Mode and manner of the election of the President of India

b) Extent of executive power of the Union

c) Extent of executive power of the States

d) Constitution of High courts for a Union Territory


pankaj

e) Provisions relating to Union Judiciary

f) Provisions relating to High Courts

g) Provisions relating to distribution of legislative powers between Center and States and
Provisions relating to Amendment

h) Provisions relating to three lists in the VII Schedule

i) Provisions relating to representation of States in Parliament85

Amendment of Basic Structure

The “basic features” principle was first expounded in 1953, by Justice J.R. Mudholkar in his
dissent, in the case of Sajjan Singh v. State of Rajasthan. The basic structure doctrine is an Indian
judicial principle that the Constitution of India has certain basic features that cannot be altered or
destroyed through amendments by the parliament. The basic features of the Constitution have not
been explicitly defined by the Judiciary, and the claim of any particular feature of the
Constitution to be a “basic” feature is determined by the Court in each case that comes before
it. Key among these “basic features”, are the fundamental rights granted to individuals by the
constitution. The basic structure doctrine does not apply to ordinary Acts of Parliament, which
must itself be in conformity with the Constitution. The present position is that the Parliament
under Article 368 can amend any part of the Constitution including the Fundamental Rights but
without affecting the ‘basic structure’ of the Constitution.

From the various judgements, the following have emerged as ‘basic features’ of the Constitution:

1. Supremacy of the Constitution

2. Sovereign, democratic and republican nature of the Indian polity


37

3. Secular character of the Constitution

4. Separation of powers between the legislature, the executive and the judiciary

5. Federal character of the Constitution

6. Unity and integrity of the nation

7. Welfare state (socio-economic justice)

8. Judicial review

9. Freedom and dignity of the individual

10. Parliamentary system


pankaj
11. Rule of law

12. Harmony and balance between Fundamental Rights and Directive Principles

13. Principle of equality

14. Free and fair elections

15. Independence of Judiciary

16. Limited power of Parliament to amend the Constitution

17. Effective access to justice

18. Principles (or essence) underlying fundamental rights.

19. Powers of the Supreme Court under Articles 32, 136, 141 and 142

20. Powers of the High Courts under Articles 226 and 227

In Kesavananda Bharati v. State of Kerala

This case was considered as the historical landmark case, where for the first-time Supreme Court
recognized the basic structure concept. In this case, the validity of the 25th Amendment was
challenged with the 24th and 29th Amendment was also questioned. The court by majority
overruled the judgement of Golaknath case. It was held that even before the 24th Amendment the
parliament has the limited power to amend the Constitution by following the proper procedure.
The Supreme Court also declared that Article 368 of the Constitution does not allow the
parliament to change, damage the basic structure of the Constitution. This landmark judgement
changes the history of the Constitution.

In Indira Nehru Gandhi v. Raj Narayan


38

Under this case, once again the basic structure concept was reaffirmed. The Supreme Court
applied the same theory and struck down the 4th clause of Article 329 A on the ground that the
Amendment is beyond the power of the parliament and it destroyed the basic structure of the
Constitution. The Amendment was made regarding the jurisdiction of all courts including the
Supreme Court, regarding the dispute of an election of the Prime Minister of India.

Qs. PERMANENT RESIDENTS

Introduction

Article 35A of the Indian Constitution was added by presidential order in 1954 on the aid and
advice of the Jawaharlal Nehru cabinet. It was promulgated in consonance with Article 370 of
the Constitution. It grants special status to J&K and empowers the Legislature of Jammu and
Kashmir to define its ‘Permanent Residents’, their rights and privileges and imposing restrictions
upon other persons pankaj

What is 35A?

Article 35A was added to the Indian Constitution by a Presidential order in 1954 – issued under
Article 370 of the Constitution. This provision allows the President to make certain “exceptions
and modifications” to the Constitution for the benefit of ‘State subjects’ of J&K. While Article
370 of the Constitution grants special status to J&K, Article 35A provides the special rights and
privileges to the permanent residents of J&K.

The Fundamental Right to Property is still guaranteed in the state. Also, certain special rights are
granted to the permanent residents of the state with regard to public employment, acquisition of
immovable property, settlement and government scholarships. It disallows people from outside
the state from buying or owning immovable property there, settle permanently, or avail
themselves of state-sponsored scholarship schemes.

Only the Jammu-Kashmir assembly can change the definition of Permanent Residents through a
law ratified by a two-thirds majority.

Who are the Permanent Residents of Jammu & Kashmir? ( section 6)

Section 6 under Part III of the Constitution of Jammu and Kashmir defines the Permanent
Residents of Jammu and Kashmir. It states that:

(1)Every citizen of India shall be a permanent resident of the State, if on the fourteenth day of
May 1954-

(a) he was a State Subject; or

(b) having lawfully acquired immovable property in the State has been ordinarily resident in
the state for not less than ten years.
39

(2) Any person who, before the fourteenth day of May 1954, was a State Subject and who had
migrated after the first day of March 1947, to the territory now included in Pakistan, returns to
the State under a permit for resettlement in the State.

Powers of state legislature to define permanent residents (section 8)

Nothing in foregoing provisions of this part shall derogate from the power of the State legislature
to make any law defining the classes the persons who are, or shall be permanent residents of the
State.

Bill Making provision (Section 9)

A Bill marking provision for any of the following matters, namely.

(a) defining or altering the definition of, the classes of persons who are, or shall be, per-manent
pankaj
residents of the State;

(b) conferring on permanent residents any special rights or privileges;

(c) regulating or modifying any special rights or privileges enjoyed by permanent residents; shall
be deemed to be passed by either House of the Legislature only if It is passed by a majority of
not less than two-thirds of the total membership of that House.

10. The permanent residents of the State shall have all the rights guaranteed to them under the
Constitution of India.

Rights and Privileges to permanent resident of J&K

1. Jammu & Kashmir; is an integral part of the Indian Union. But its area, name and boundary
can’t be altered without the consent of the state assembly.

2. According to this article, the central government has to get approval from the state
government to implement all other laws except defense, foreign affairs and communication in the
state.

3. Jammu and Kashmir has its constitution because of article 370 and its administration is run
accordingly not according to the Constitution of India.

4. J & K has 2 flags; One of Kashmir and another is India's Tricolor flag.

5. The citizens of other Indian states can not buy any property or kind of any property in this
state. It means, the fundamental right to property is still in force in this state.

6. The people of Jammu and Kashmir have two types of citizenship. One is Indian citizenship
and another is Kashmiri citizenship. Worth to mention that no other Indian can have two
citizenship simultaneously.
40

7. If a Kashmiri woman marries an Indian, then her Kashmiri citizenship terminates, but if she
marries a Pakistani, it does not affect her citizenship status.

8. If a Pakistani boy marries a Kashmiri girl, he gets Indian citizenship too while Indians don’t
have this privilege.

10. Part 4 of the Indian Constitution (Directive Principal of State Policy) and Part 4A
(Fundamental Duties) are not applicable in this State.

It means the citizens of Kashmir are not bound to save the cow, maintain the dignity of the
women and respect the National Flag of India.

11. One of the most shocking right is that Insulting National Symbols of India (National Anthem,
National Flag etc.) in J & K does not fall under the category of crime.
pankaj
12. The President of India does not have power to declare financial emergency in the state.

13. Any amendment in the Constitution of India does not apply automatically to J & K unless a
special order of President is not passed.

14. The Central government can impose National Emergency in the state in two conditions only;
war and external invasion.

15. The President has no power to suspend the constitution of the state on the ground of failure to
comply with the directions given by him.

16. If the national emergency is imposed in the country on the basis of internal disturbance; this
emergency is not applicable in the Jammu and Kashmir until it is approved by the state
government.

17. The Central Government can not impose National Emergency in the state on the basis of
internal disturbance in the state. The central government must take the permission of state
government before doing so.

18. Only the resident of the Kashmir can take selection in the jobs of the state government.
41

Unit-IV

Qs. Doctrine of Pleasure – Art. 310, Protection Against arbitrary dismissal – Art.311 and
exceptions.

Ans. The doctrine of pleasure owes its origin to common law. The rule in England was that a
civil servant can hold his office during the pleasure of the crown and the service will be
terminated any time the crown wishes the same rule is applied in India. The member of Defence
services or civil services of the union or All-India services hold their office during the pleasure
pankaj
of president. Similarly member of state services holds the office during the pleasure of governor.
the provisions related to services under union and state is contained under part XIV of the Indian
constitution.

The article 310 of Indian constitution

Except as expressly provided by this Constitution, every person who is a member of a Defence
service or of a civil service of the Union or of an All India Service or holds any post connected
with Defence or any civil post under the Union holds office during the pleasure of the President,
and every person who is a member of a civil service of a State or holds any civil post under a
State holds office during the pleasure of the Governor of the State.

Now if such powers are given to president of India and the governor of states than it would be
really difficult to exercise power on them so there are certain offices which are outside the
purview of article 310 and article 311 was put as a restriction to doctrine of pleasure.

Services excluded from the purview of Article 310

1.Tenure of supreme court judges{Article124}

2.Tenure of high court judges{Article148(2)}

3.The chief election commissioner{Article324}

4.Chairman and member of public- service commission{Article317)

The article 311 acts as a safeguard to civil servants

(1) No person who is a member of a civil service of the Union or an all-India service or a civil
service of a State or holds a civil post under the Union or a State shall be dismissed or removed
by an authority subordinate to that by which he was appointed.
42

(2) No such person as aforesaid shall be dismissed or removed or reduced in rank except after an
inquiry in which he has been informed of the charges against him and given a reasonable
opportunity of being heard in respect of those charges: Provided that where, it is proposed after
such inquiry, to impose upon him any such penalty, such penalty may be imposed on the basis of
the evidence adduced during such inquiry and it shall not be necessary to give such person any
opportunity of making representation on the penalty proposed: Provided further that this clause
shall not apply —

(a) where a person is dismissed or removed or reduced in rank on the ground of conduct which
has led to his conviction on a criminal charge; or

(b) where the authority empowered to dismiss or remove a person or to reduce him in rank is
satisfied that for some reason, to be recorded by that authority in writing, it is not reasonably
practicable to hold such inquiry; or
pankaj

(c) where the President or the Governor, as the case may be, is satisfied that in the interest of the
security of the State it is not expedient to hold such inquiry.

(3) If, in respect of any such person as aforesaid, a question arises whether it is reasonably
practicable to hold such inquiry as is referred to in clause (2), the decision thereon of the
authority empowered to dismiss or remove such person or to reduce him in rank shall be final."

The procedure laid down in Article 311 is intended to assure, first, a measure of security of
tenure to Government servants, who are covered by the Article and secondly to provide certain
safeguards against arbitrary dismissal or removal of a Government servant or reduction to a
lower rank. These provisions are enforceable in a court of law. Where there is an infringement of
Article 311, the orders passed by the disciplinary authority are void ab-initio and in the eye of
law "no more than a piece of waste paper" and the Government servant will be deemed to have
continued in service or in the case of reduction in rank, in his previous post throughout. Article
311 is of the nature of a proviso to Article 310. The exercise of pleasure by the President under
Article 310 is thus controlled and regulated by the provisions of Article 311.

When Article 311 is applicable.

The most notable point is that Article 311 is available only when ` dismissal, removal, reduction
in rank is by way of punishment `.so it is difficult to determine as to when an order of
termination of service or reduction in rank amounts to punishment

in case of Parshottam Lal Dhingra Vs Union of India. The supreme court laid down 2 tests to
determine when termination is by way of punishment –

I. Whether the servant had a right to hold the post or the rank;
43

II. Whether he has been visited with evil consequences. If a government servant had a right
to hold the post or rank under the terms of any contract of service, or under any rule,
governing the service, then the termination of his service or reduction in rank amounts to
a punishment and he will be entitled to protection under Article 311. Articles 310 and 311
apply to Government servants, whether permanent, temporary, officiating or on
probation.

Exceptions to Article 311(2)

The provision to Article 311 (2) provides for certain circumstances in which the procedure
envisaged in the substantive part of the clause need not be followed. These are set out below.
1. Conviction on a criminal charge- One of the circumstances excepted by clause (a) of the
provision is when a person is dismissed orremoved or reduced in rank on the ground of conduct
which has laid to his conviction on a criminal charge. The rationale behind this exception is that
pankaj
a formal inquiry is not necessary in a case in which a court of law has already given a verdict.
However, if a conviction is set aside or quashed by a higher court on appeal, the Government
servant will be deemed not to have been convicted at all. Then the Government servant will be
treated as if he had not been convicted at all and as if the order of dismissal was never in
existence. In such a case the Government servant will also be entitled to claim salary for the
intervening period during which the dismissal order was in force. The claim for such arrears of
salary will arise only on reinstatement and therefore the period of limitation under clause 102 of
the Limitation Act would apply only with reference to that date.

The grounds of conduct for which action could be taken under this proviso could relate to a
conviction on a criminal charge before appointment to Government service of the person
concerned. If the appointing authority were aware of the conviction before he was appointed, it
might well be expected to refuse to appoint such a person but if for some reason thefact of
conviction did not become known till after his appointment, the person concerned could be
discharged from service on the basis of his conviction under clause (a) of the proviso without
following the normal procedure envisaged in Article 311.

2. Impracticability- Clause (b) of the proviso provides that where the appropriate disciplinary
authority is satisfied, for reasons to be recorded by that authority in writing that it does not
consider it reasonably practicable to give to the person an opportunity of showing cause, no such
opportunity need be given. The satisfaction under this clause has to be of the disciplinary
authority who has the power to dismiss, remove or reduce the Government servant in rank. As a
check against an arbitraryuse of this exception, it has been provided that the reasons for which
the competent authority decides to do away with the prescribed procedures must be recorded in
writing setting out why it would not be practicable to give the accused an opportunity. The use of
this exception could be made in case, where, for example a person concerned has absconded or
where, for other reasons, it is impracticable to communicate with him.
44

3. Reasons of security- Under proviso (c) to Article 311 (2), where the President is satisfied that
the retention of a person in public service is prejudicial to the security of the State, his services
can be terminated without recourse to the normal procedure prescribed in Article 311 (2).The
satisfaction referred to in the proviso is the subjective satisfaction of the President about the
expediency of not giving an opportunity to the employee concerned in the interest of the security
of the State. This clause does not require that reasons for the satisfaction should be recorded in
writing. That indicates that the power given to the President is unfettered and cannot be made a
justifiable issue, as that would amount to substituting the satisfaction of the court in place of the
satisfaction of the President.

Is suspension or compulsory retirement a form of punishment?

Neither suspension nor compulsory retirement amounts to punishment and hence they can't be
brought under the purview of Article311 and has no protection is available.
pankaj

Supreme court in case of such Bansh singh Vs State of Punjab clearly held that suspension from
service is neither dismissal nor removal nor reduction in rank, therefore, if a Government servant
is suspended he cannot claim the constitutional guarantee of Article 311[2]

In Shyam Lal Vs State of U.P Supreme Court held that compulsory retirement differ from
dismissal and removal as it involves no penal consequences and also a government servant who
is compulsory retired does not loose any part of benefit earned during the service so it doesn't
attract the provisions of Article 311.

Qs Anti defection Law including position under J & K Constitution.

What is Anti-Defection Law?

Before defining anti-defection law, we should understand the meaning of defection. When an
elected representative joins another party without resigning his present party for benefits, it is
called defection. Thus a defector is one who is elected from one party and enjoys power in
another party.

The word defection is also called as “Floor Crossing” in UK and “Carpet Crossing” in Nigeria.
In India, the term used for this is “Defection”. Defection is commonly known as “Horse
Trading”. Defectors are also called “Fence Sitters” or “Turn Coats”.

Background

Rajiv Gandhi, the then Prime Minister of India, proposed a bill to remove the evils of defection.
The Parliament passed the bill as a result of which anti-defection act came into force on 1st April
1985 through 52nd Constitutional Amendment.

52nd Amendment
45

The 52nd amendment to the Constitution laid down rules and procedures for restricting members
of Parliament and State Legislatures from defecting from one party to the other at their sweet
will. For this purpose a new schedule, known as the tenth schedule, was incorporated in the
Constitution. Through this, the process by which legislators may be disqualified on grounds of
defection was laid down in detail.

Provisions

The main intention of the law is to combat “the evil of political defections”. The provisions are:

1. A member of Parliament or State Legislature belonging to any political party shall be


disqualified if he quits his party at his own will.
2. He will be disqualified from his membership if he votes against his party Whip or
abstains from voting.
3. If the member takes prior permission for pankajvoting or abstention from the party within 15

days from such voting or abstention, he/she shall not be disqualified.


4. A member of Parliament or State Legislature belonging to any political party shall be
disqualified from his membership if he votes in the session without prior permission of
his party.
5. A nominated member shall be disqualified from his membership in the Upper House if he
joins any political party after six months from the date on which he assumes his position.
6. If 2/3rd strength of any political party merges with another political party it shall not be
considered as defection.
7. A person disqualified under this Act shall not be provided any office of profit.
8. The anti-defection law determines the size of the Council of Ministers. The size of the
Council of Ministers shall not exceed 15% of the total members of Lok Sabha and State
Legislative Assemblies.
9. Speaker can start action against the members under anti-defection law.
10. The Chairperson of Legislatures are permitted to frame the rules to implement this law.
11. An independent member of the house shall be disqualified, if he joins any party after
defection.

Some other provisions

I. A person shall not be disqualified if his party merges with another party under this law.
II. The Speaker and Deputy Speaker of Lok Sabha and Deputy Chairman of Rajya Sabha,
the presiding officers of State Legislature shall not be disqualified under Anti-Defection
law if they quit their original party.
III. A person who is disqualified under anti-defection law can challenge the orders of the
Speaker in the Court.

However, under the following circumstances, a split in a party not considered a 'defection':
46

a. If an entire political party merges with another


b. If a new political party is formed by some of the elected members of one party
c. if he or she or other members of the party have not accepted the merger between
the two parties and opted to function as a separate group from the time of such a
merger.

Qs. Supreme Court of India and its original appellate and advisory jurisdiction

Supreme Court at the apex of Indian Judiciary is the highest authority to uphold the constitution
of India, to protect rights and liberties of citizens and to uphold the values of rule of law. Hence
it is known as the guardian of our Constitution.

The Indian constitution provides for a provision of Supreme Court under Part 5 (The Union) and
Chapter 6 titled The Union Judiciary. Indian Constitution has provided an independent judiciary
pankaj
with a hierarchical setup containing High Courts and Subordinate Courts under it.

Eligibility to become a Supreme Court judge

To become a judge of the Supreme court, an individual should be an Indian citizen. The norms
relating to the eligibility has been envisaged in the Article 124 of the Indian Constitution.

In terms of age, a person should not exceed 65 years of age. The article further laid out that to
become elevated as a judge of the top court,

a person should serve as a judge of one high court or more (continuously), for at least five years

or the person should be an advocate in the High court or the Supreme court for at least 10 years
or a distinguished jurist.

Composition of the Supreme Court

Article 124(1) and Amendment act of 2008 states that there shall be a Supreme Court of
Indiaconsisting of a Chief justice of India (CJI) and 31 judges including the CJI. Article 124(2)
states that every judge of the Supreme Court shall be appointed by the President by warrant
under his hand and seal after consultation with such of the judges of Supreme Court and of the
High Courts in the states.

Salary

The Chief Justice of India will now get a monthly salary of Rs2.80 lakh, up from the present Rs1
lakh. Similarly, judges of the Supreme Court and chief justices of high courts will draw a
monthly salary of Rs2.50 lakh, up from the current Rs90,000, according to the act notified by the
law ministry.

The judges of high courts, who get Rs80,000 per month now, will get Rs2.25 lakh per month
47

Tenure

A Judge of the Supreme Court may cease to be so on the happening of any one of the following
contingencies-

(1) On attaining the age of sixty five years;

(2) On resigning his office by writing under his hand addressed to the President;

(3) On being removed from his office by an order of the President passed after an address by
each House of Parliament supported by a majority of not less than two-thirds of the members of
that House present and voting and has been presented to the President in the same session of
such removal on the ground of proved misbehaviour or incapacity.

Appointment of ad hoc judges-


pankaj

Art 127 states that if at any time there is lack of quorum of Judges of Supreme Court, the CJI
may with previous consent of the President and Chief Justice of High Court concerned request in
writing the attendance of Judge of High Court duly qualified to be appointed as Judge of SC.

Apointment of acting Chief Justice- Art 126-

when the office of CJI is vacant or when the Chief Justice is by reason of absence or otherwise
unable to perform duties of office, the President in such case can appoint Judge of the court to
discharge the duties of office.

Removal of Supreme Court Judge:

A judge of Supreme Court can be removed only from the office by the President on the basis of a
resolution passed by both the Houses of parliament with a majority of the total membership and a
majority of not less than two-thirds of the members present and voting in each house, on the
grounds of proved misbehaviour or incapacity of the judge in question.

Hence, a democratic country like India needs a judiciary because democratic values tend to lose
their prominence without proper checks and balances

Jurisdiction

The Supreme Court of India has three kind of jurisdiction-

(1) Original,

(2) Appellate, and

(3) Advisory.
48

(1) Original Jurisdiction Art. 131- The Supreme Court shall, to the exclusion of any other
court, have original jurisdiction in any dispute-

(a) Between the Government of India and one or more States; or

(b) Between the Government of India and any State or States on one side and one or more other
States on the other; or

(c) Between two or more States.

(2) Appellate Jurisdiction – Art. 132The Appellate Jurisdiction of the Supreme Court extends
to four heads

(i) Civil,

(ii) Criminal, pankaj

(iii) Constitutional, and

(iv) Special leave.

(i) Civil appellate jurisdiction - An appeal shall lie to the Supreme Court from any judgment,
decree or final order in a civil proceedings of a High Court in the territory of India if the High
Court certifies under Article 134-A-

(a) that the case involves a substantial question of law of general importance; and
(b) that in the opinion of the High Court the said question need to be decided by the Supreme
Court.

(ii) Criminal appellate jurisdiction - An appeal shall lie to the Supreme Court from any
judgement, final order or sentence in criminal proceedings of a High Court in the territory of
India if the High Court-

(a) has on appeal reversed an order of acquittal of an accused person and sentenced him to death;
or

(b) has withdrawn for trial before itself any case from any court subordinate to its authority and
has in such trial convicted the accused person and sentenced him to death; or

(c) certifies under Article 134-A that the case is a fit one for appeal to the Supreme Court:

(iii) Constitutional appellate jurisdiction - An appeal shall lie to the Supreme Court from any
judgement, decree or final order of a High Court in the territory of India whether in a civil,
criminal or other proceeding, if the High Court certifies under Article 134-A that the case
involves a substantial question of law as the interpretation of this Constitution.Where such
49

certificate is given, any party in the case may appeal to the Supreme Court on the ground that
any such question has been wrongly decided.

(iv) Special leave - Supreme Court may, in its discretion, grant special leave to appeal from any
judgement, decree, determination, sentence or order in the territory of India. It will not apply to
any judgement, determination, sentence or order passed or made by any Court or tribunal
constituted by or under any law relating to the Armed Forces.

(3) Advisory Jurisdiction – Art. 143 If at any time it appears to the President that a question of
a law or fact has arisen or is likely to arise, which is of such a nature and such public importance
that it is expedient to obtain the opinion of the Supreme Court for consideration and the Supreme
Court may refer the question to the Supreme Court for consideration and the Supreme Court
may, after such hearing as it think fit, report to the President its opinion thereon.
The President may refer a dispute to the Supreme Court for opinion and the Supreme Court shall,
pankaj
after such hearing as it thinks fit, report to the President its opinion thereon.

Qs. High Court and its original appellate and advisory jurisdiction.

Appointment of the Judges of High Courts

The procedure of appointing the Judges of the High Courts in India is slightly different from the
appointment of the Judges of the Supreme Court. As per article 217, the chief Justice of the high
court is appointed by the President in consultation with the Chief justice of India as well as the
Governor of the state in question. A collegium system has evolved over the years in which a
Collegium headed by the CJI makes recommendation to the government for appointment of
judges. The Collegium recommends the names to the law ministry which after scrutinizing send
the paper to the president. The president either approves the names or returns the names for
reconsideration of the Supreme Court. If still the Supreme Court sends the same names president
appoints the persons recommended.

Qualification to Become a High Court Judge

I. A person to be appointed as a judge of a high court, should be a citizen of India. Further,


II. He should have held a judicial office in the territory of India for ten years or
III. should have been an advocate of high court(s) for ten years.
IV. There is no minimum age fixed for high Court judges, and unlike in Supreme Court, there
is no provision for appointment of a distinguished jurist as a judge of a high court.

Term and Salary

A Judge of High Court holds the office until he completes the age of 62 years. (In Supreme
Court it is 65 years). The salaries and allowances of the Chief Justice of High Court and Judges
of the High Court are decided by the parliament by law, time to time.
50

Current salary of Chief Justice is Rs. 90,000

Current salary of other judges is Rs. 80, 000

The salaries and other expenses of the judges and maintenance of the state high courts are
charged from consolidated fund of the state. Pension of retired high court judges comes from
Consolidated Fund of India.

Removal of the Judge of a High Court

A Judge of the High Court can be removed from office only for proven misbehaviour or
incapacity and only in the same manner in which a Judge of the Supreme Court is removed. The
President of India can remove a Judge of the High Court, from his office only if each house of
the parliament passes a resolution by a two third majority of its members present and voting in
each house requesting him to remove the Judge.pankaj

Transfer of Judges

Transfer of High Court Judges is done by the President in consultation with the following-

I. Chief justice of India’ whose opinion is formed by senior most judges of the Supreme
Court.
II. Chief Justice of the High court from where transfer is to take place.
III. Chief Justice of the High Court to where the transfer is to take place

Post retirement Jobs

The retired permanent judges of a high court are prohibited from pleading or acting in any court
or before any authority in India except the Supreme Court and the other high
courts. However, government generally uses the retired higher judiciary judges as heads of
various commissions. There has been a demand from certain sections of the society that there
should be a “cool off” period of two years for the retired judges before they are installed in other
offices.

Jurisdiction, Powers and Control of High Court

The High Court is vested with extensive powers such as highest court of appeal in concerned
state, protector of fundamental rights (FRs) of citizens or power to interpretation of the
constitution. Also High Court has supervisory and advisory powers over lower courts or tribunal
in concerned state.

Original Jurisdiction

The High Court has original or exclusive jurisdiction (i.e. to hear in first stance) in matters that
are as follows –
51

I. Matters of admiralty, will, marriage, divorce, company laws and contempt of court.
II. Any election dispute in relation to Parliament and State legislature;
III. Disputes in relation to revenue matters;
IV. Enforcement of FRs of citizens;
V. Any matter in relation to interpretation of constitution pending in lower courts;
VI. The four high courts (Delhi, Bombay, Calcutta and Madras) have original civil
jurisdiction in cases of higher value.
VII. Till 1973, 3 high courts (Calcutta, Bombay and Madras) had original criminal jurisdiction
but abolished by CrPC, 1973.

Writ Jurisdiction

The Writ Jurisdiction of High Court is provided under Article 32 of the Constitution. The
constitutional writs such as Habeas Corpus, Mandamus, Prohibition, Quo Warranto or
pankaj
Certiorari are tools for enforcement of Fundamental Rights and for any other purpose.

“Any other purpose” includes any ordinary legal right of a citizen. So High Court can issue writs
to any person, authority and govt for enforcement of an ordinary right of a citizen. This power to
issue writ is not limited to state where seat of High Court resides but can extend to other state or
territory if cause of action arises in its territorial jurisdiction.

The aggrieved party in relation to matters of FRs can directly approach to High Court and SC. So
High Court has wider writ jurisdiction in relation to SC (as SC can issue writ only for
enforcement of FRs and not for any ordinary legal right as of High court).

In Chandra Kumar Case (1997), SC held that Writ Jurisdiction of SC and High Court constitutes
a basic structure of the constitution and can’t be taken away even by an amendment to the
constitution.

Appellate Jurisdiction

The High Court is the highest court of appeal in concerned State and enjoys wide appellate
jurisdiction in cases:-

Civil Matters

First appeal from orders of dist, additional dist and other subordinate courts on both questions of
law and fact if amount of fine exceed a pre defined limit.

Second appeal from orders of dist, additional dist and other subordinate courts only on question
of law (not includes fact).

The Calcutta, Bombay and Madras High Courts have provision for intra court appeal. It means if
any judgement or order is delivered by single judge bench then appeal for such case lies to
division bench of same high court.
52

Appeals from administrative and other quasi judicial tribunals lie to division bench of the state
High court. In 1997, SC ruled that tribunals are subjected to writ jurisdiction of High Court.
So an aggrieved person by decision of tribunals has to approach State High Court first rather than
SC directly.

Criminal Matters

Appeals from order or decision of session and additional session court judge lie to High Court if
sentence of imprisonment is more than 7 yrs. But death sentence shall get approval of High
Court whether an appeal made by convict or not.

Appeals from decision of assistant session, metropolitan or other judges lie to High Court in
some specified cases other than petty cases as under section 374, 376, 376G of CrPC (1973).

Supervisory Jurisdiction (Article 227) pankaj

Every High Court has power of superintendence of all courts and tribunals which falls within its
territorial jurisdiction except military courts or tribunal and High Court may

I. call for returns from such courts;


II. make and issue general rules and prescribe forms of regulating the practice and
proceedings of such courts;
III. prescribe forms in which books, entries and accounts shall be kept by officers of any such
court.

The supervisory power of High Court is very wide in nature as

I. It extends to all courts or tribunals whether they lies under appellate jurisdiction of High
Court or not;
II. It includes revisional jurisdiction to intervene in cases of gross injustice or abuse of
jurisdiction or non exercise/refusal of jurisdiction or in cases of an error of law apparent
on faces of the record or violation of principles of natural justice or arbitrary exercise of
authority or findings are on no material/fact or error in procedure;
III. It includes both administrative and judicial superintendence;
IV. It can be exercised Suo Motto (on its own) either an application filed by party or not.
V. Control over subordinate courts
VI. As a head of judiciary in concerned state, the High Court has administrative control over
sub ordinate courts and these are as follows:-
VII. The High Court shall be consulted by governor in matters of appointment, posting and
promotion of district judges (Article 233).
VIII. The High Court is to be consulted along with State PSC by governor in appointing
persons to the judicial service of the state other than dist judges (Article 234).
53

IX. The High Court can withdraw any case pending in lower courts if it involves ample
question of law that requires interpretation of the constitution.

Court of Record

The judgement, proceedings and acts of High Court are recorded and enduring in nature. These
records are treated as evident and can’t be questioned when produces before any lower court.
These act as legal references. The High Court has power to punish for contempt of court either
with simple imprisonment or fine or both. As a court of record, High Court has power to review
its own decision or orders albeit any such power not confers by constitution.

Judicial Review

The High Court has power of judicial review over all legislative and executive acts of central and
state govts. If these acts found in violation of provisions
pankaj of the constitution then can be declared
null & void by High Court. The validity of legislative and executive acts can be challenged in
High Court on grounds which are as follows:-

I. It is in conflict with or violation of any of FRs that explained under Part III.
II. If such act is not under competence of authority which has framed it;
III. If such act is in contradiction with constitutional provisions.

You might also like